Principio de acción para el Electromagnetismo y la Gravedad

Aquí está la fórmula para el tensor de energía de estrés:

T m v = 2 | det gramo | d S mi METRO d gramo m v
(Esto se sigue de variar la acción total S = S H + S mi METRO , dónde S H = C 4 dieciséis π GRAMO R | det gramo m v | d 4 X es la acción de Hilbert y da las ecuaciones de Einstein, y S mi METRO son otros términos en el Lagrangiano, que contribuyen al lado derecho de las ecuaciones de Einstein en forma de T m v arriba.) El Lagrangiano del campo electromagnético es:

S mi METRO 1 = 1 4 m 0 F α β F α β | det gramo | d 4 X

y usando la fórmula anterior, obtenemos para el tensor de energía de tensión:

T m v = 1 m 0 ( F m β F v β 1 4 F α β F α β gramo m v )
cual es el elmag correcto. tensor de energía de tensión. Sin embargo, la parte de interacción del archivo elmag. Lagrangiano es
S mi METRO 2 = j m A m | det gramo | d 4 X
y si interpretamos esto como una función de gramo m v también contribuiría al tensor de energía de estrés de esta manera:
d S mi METRO 2 = d j m A m | det gramo | d 4 X = d ( gramo m v j m A v | det gramo | ) d 4 X =
= ( d gramo m v ) ( j m | det gramo | ) ) A v d 4 X

donde el j m | det gramo | se trata como la densidad de corriente (y por lo tanto no depende de gramo m v al variar), sin embargo, claramente el tensor de energía de esfuerzos correspondiente a esto sería:

T m v = 2 j m A v
(Posiblemente solo contribuye la parte simétrica, porque la parte antisimétrica se cancela con gramo m v , por lo que obtendríamos T m v = j m A v + j v A m .) En cualquier caso, dichos términos deberían aparecer en el lado derecho de las ecuaciones de Einstein. Sin embargo, no creo que esto sea correcto.

¿Alguien sabe lo que está mal aquí?

Las variables que estás variando son A m y gramo m v . Desde A m aparece naturalmente con un índice reducido, la variación de j m A m con respecto a la métrica inversa es cero, a menos que existan factores de gramo m v escondido en el j m . Además, te estás perdiendo el factor de R eso te da la dinámica de la gravedad, aunque eso podría ser intencional.
Hola Jerry, creo que esto es todo. (Sí, me salté el factor R para las ecuaciones de Einstein aquí.) Pero, ¿cómo sé que a veces necesito usar el gramo m v factor (y luego variarlo), y a veces no lo hago? Me parece que es completamente arbitrario.
Tienes que tomar una decisión sobre qué término vas a variar. No hay factor de gramo m v en A m j m , porque A m aparece naturalmente con un índice hacia abajo. Ese término es una forma que actúa sobre un vector, lo que sucede independientemente de la métrica. Y j m es naturalmente un vector y no una forma, porque normalmente se define por j m d L C h a r gramo mi d metro a t t mi r d A m .
Entonces, por lo tanto, en el sentido natural, no hay dependencia de la métrica en ninguno de los dos términos.
Jerry, muchas gracias. Lo que escribes tiene mucho sentido para mí. Pero ¿qué pasa con un término como pag m pag m (ver physics.stackexchange.com/questions/17604/… )? ¿Será que por ser un término mixto con pag m y pag m uno tiene que convertirlo para usar solo pag m (digamos) y uno obtiene gramo m v pag m pag v y la métrica está (¿naturalmente?) ahí. Mientras que para un término como j m A m = j m A m , la métrica no está naturalmente allí. Pero supongo que realmente tengo que asumir (especificar) esto de antemano, ¿verdad?
Una pregunta más: ¿por qué A m aparecen naturalmente con el índice más bajo? ¿Es así que F m v = m A v v A m tiene ambos índices hacia abajo? (La derivada naturalmente tiene un índice a la baja, y uno necesita usar la métrica para aumentarla, eso me queda claro).
@Ondrej: Obtiene la misma fuente de tensor de estrés para las ecuaciones de Einstein con cualquier posición de índice, como describo en mi respuesta. El índice más bajo da una variación un poco más simple, eso es todo.
@Ondrej: pero tienes que decidir variar una opción u otra. Una vez que haya arreglado eso, habrá decidido qué grados de libertad son de campo y cuáles se componen de operadores de inversión y operaciones de elevación, etc.
Puede encontrar útil la Sección 48 de 'La teoría del espacio, el tiempo y la gravitación', de VA Fock.

Respuestas (3)

Los comentarios de Jerry Schirmer te dicen la idea principal, pero me gustaría darlos más explícitamente y en forma de respuesta. Cuando varías la acción con respecto a A m , la variación métrica no da el término no deseado que mencionas. Pero si varías con respecto a A m , obtienes este término, y no debería aparecer en el lado derecho de las ecuaciones de Einstein, dado que ya conocemos esas ecuaciones de la versión A 1-forma de la variación.

Pero a las ecuaciones de movimiento no debería importarles si eliges variar con respecto a A m o con respecto a A m , debe obtener las mismas ecuaciones. Formalmente

d S = d S d gramo m v d gramo m v + d S d A m d A m

Y las ecuaciones de Einstein son los coeficientes de d gramo , mientras que las ecuaciones de Maxwell (potencial vectorial no vacío) son los coeficientes de d A .

Las variaciones en A m , gramo m v puede expresarse fácilmente en términos de las variaciones en A m , gramo m v ,

d A v = d A m gramo m v + A m d gramo m v

Lo cual, al expresar la variación total de la acción, mezcla linealmente las partes de Einstein y Maxwell:

d S = ( d S d gramo m v + d S d A m A v ) d gramo m v + d S d A m gramo m v d A v

Donde las derivadas variacionales son todas las antiguas derivadas variacionales, con respecto al par gramo m v , A m sosteniendo el otro fijo. Estas combinaciones lineales dan las nuevas variaciones. Es trivial ver que las nuevas ecuaciones de movimiento se satisfacen si y solo si se cumplen las antiguas, por lo que nada ha cambiado.

Las nuevas ecuaciones de Maxwell son, después de multiplicarlas por la métrica inversa, las mismas que las antiguas. Pero la nueva ecuación de Einstein tiene un término fuente adicional:

d S d A m A v

Este término fuente adicional es obviamente cero, según las ecuaciones de Maxwell, pero d S d A m incluye el término j m , entonces el término que te estaba molestando aparece aquí. Esta variación da un lado derecho de las ecuaciones de Einstein que incluye una tensión adicional que incluye el término fuente, en la forma de la ecuación de Maxwell multiplicada por el vector potencial.

( D m F m v j v ) A m

Pero ahora es obvio que la contribución del estrés se desvanece (como siempre lo fue, porque esto es solo una variación con respecto a diferentes variables de la misma acción).

Sobre las variaciones de densidad

La respuesta de Jon calcula un término adicional a partir de la variación gramo , pero este término no está presente. Esto es por la razón explicada en la respuesta a esta pregunta: Lagrangian para preguntas de derivación de polvo relativista .

Cuando varía la métrica con EM y, digamos, una fuente de polvo cargada, mantiene j gramo fijado. Esto es por la misma razón por la que la densidad de momento se mantiene fija, usted mantiene constante el número de líneas de mundo cuando varía g, de modo que las corrientes y cargas conservadas se conservan bajo variaciones métricas.

¡Gracias! Estaba pensando en su respuesta en "Lagrangian para preguntas de derivación de polvo relativista" y ahora me queda claro que debe mantener j gramo fijado. Por lo tanto, no obtendrá términos adicionales derivados de la variación gramo , eso está claro ahora. Pensaré en el resto de su respuesta durante un tiempo (y lo resolveré yo mismo para entenderlo bien) antes de aceptarlo.
Corrección: d A norte tu arriba debe ser d ( A m gramo m v ) .
@Ondrej: sí, eso fue un error tipográfico para d A v fijado.

Creo que, en la expresión

S mi METRO 2 = j m A m gramo d 4 X
tienes que derivar también el determinante. Si lo haces obtendrás
1 2 gramo ( j v A m + j m A v ) d gramo m v 1 2 gramo m v gramo j A d gramo m v .
Entonces, obtendrás
T m v ( 2 ) = 2 gramo d S mi METRO 2 d gramo m v = ( j v A m + j m A v ) + gramo m v j A .
Este debería ser el resultado correcto.

La fórmula de la energía de tensión es T m v ( 2 ) = 2 gramo d S mi METRO 2 d gramo m v para que no entiendas 1 2 en la respuesta final.
Si el determinante también necesita variarse, entonces su derivación es correcta (aunque Dirac no la varía). Sin embargo, nunca antes había visto tal tensor de energía de estrés (compuesto de j y A ). Solo T m v = 1 m 0 ( F m β F v β 1 4 F α β F α β gramo m v ) . ¿Tiene algún puntero en la literatura sobre esto? ¿Cuál es su interpretación física?
Comentario menor: creo que te equivocaste en el signo general, hay menos en la acción, menos en la definición del tensor de energía de tensión y menos al variar el determinante, así que creo que la respuesta correcta es
T m v ( 2 ) = 2 gramo d S mi METRO 2 d gramo m v = + ( j v A m + j m A v ) gramo m v j A .
. Pero de todos modos, el signo no es importante para esta discusión.
Gracias, he arreglado 1/2 factor. Puede consultar aquí en.wikipedia.org/wiki/Maxwell%27s_equations_in_curved_spacetime . El término de interacción da una contribución a la rhs de las ecuaciones de Einstein. Esto puede entenderse como que, cuando hay materia cargada, se suma una contribución al tensor de energía de tensión.
Gracias. Entonces, solo para que nos entendamos, está diciendo que el lado derecho de las ecuaciones de Einstein contendrá: T m v ( 1 ) + T m v ( 2 ) , dónde T m v ( 1 ) = 1 m 0 ( F m β F v β 1 4 F α β F α β gramo m v ) y T m v ( 2 ) = ( j v A m + j m A v ) + gramo m v j A . Fui a la wiki, y puedo ver allí el T m v ( 1 ) término, pero no el T m v ( 2 ) término. ¿Me estoy perdiendo de algo?
Eso es lo que quiero decir. La materia también se puede cargar y esto debe tenerse en cuenta en las ecuaciones de Einstein.
Gracias por la aclaración. tengo dudas sobre el T m v ( 2 ) término: nunca lo he visto (avíseme si me lo perdí en el wiki). En realidad, no importa si varía el determinante o no (solo agrega el j A término). La materia cargada influye en el elmag. campo mismo ( A m resp. F m v ) que forma parte del T m v ( 1 ) tensor, la materia de carga se contabiliza mediante el F m v campo ya. Pero es confuso para mí, por qué no existe tal término, porque parece que el principio variacional lo sugiere.
No he entendido del todo tus últimas frases. Si tiene materia cargada, por ejemplo, un objeto esférico cargado, debe tener esto en cuenta en la ecuación de Einstein y no está contenido en el tensor de energía de estrés em. Puedes entender mejor esto si consideras que la derivada covariante de la rhs de las ecuaciones de Einstein te dará las ecuaciones de movimiento para el asunto. tienes que imponer m T m v = 0 .
Jon, según tengo entendido, la materia cargada genera el campo elmag. F m v a través del lado derecho de las ecuaciones de Maxwell (que contiene j m ). Como tal, la materia cargada genera F m v . Este F m v ya es parte de T m v ( 1 ) , y como tal, ya se tiene en cuenta en las ecuaciones de Einstein. En otras palabras, la materia cargada curva el espacio-tiempo debido a su masa y también debido a su carga/corrientes. Si existe o no también el término T m v ( 2 ) es un tema diferente. Si conoce alguna literatura o página web sobre esto, por favor indíqueme.

En realidad, su fórmula inicial es incorrecta: el d ( S mi METRO ) / d ( gramo ( tu , v ) ) debiera ser d ( L mi METRO / d ( gramo ( tu , v ) ) , dónde L (el lagrangiano) es lo que se integra en el espacio para obtener la acción S .

Es una cosa de manzanas y naranjas: la acción es la integral del Lagrangiano, d (acción) sigue siendo un número, no una función espacial como se requiere.

Curiosamente, el libro de Wald y el libro de MTW tienen este error, al igual que usted, por lo que es comprensible. Si observa otras fuentes sobre la teoría del campo de Lagrange, o incluso el artículo de Wikipedia sobre el tensor de energía y estrés bajo el tensor de energía y estrés de Hilbert, lo tienen correcto, tomando delta de la función de densidad L en lugar de su integral, S, al definir el tensor tensión-energía (que es una función espacial).

Esto no tiene ningún efecto en su resolución del problema, pero es notable que estos dos libros de texto ampliamente utilizados sobre GR escriban esto que no tiene sentido y todos simplemente lo aceptan y proceden como si tuviera sentido. Los estudiantes son personas muy tolerantes.

¿Puedes ser más específico y usar la notación Latex para escribir cuál crees que debería ser la fórmula correcta? Creo que la forma en que lo escribí es correcta. Toma la densidad lagrangiana , integra sobre el espacio-tiempo (integral 4D) para obtener la acción. Luego aplicas la derivada funcional a la acción , no la densidad lagrangiana. Esa es al menos la definición estándar, que yo sepa.